Đến nội dung

Nguyenhuyen_AG

Nguyenhuyen_AG

Đăng ký: 09-09-2010
Offline Đăng nhập: 10-01-2019 - 16:22
****-

#532400 [Chuẩn hóa] $\frac{(2a+b+c)^2}{2a^2+(b+c)^2}+...

Gửi bởi Nguyenhuyen_AG trong 08-11-2014 - 20:28

còn việc chuẩn hóa là dành cho bđt đối xứng và thuần nhất

bđt đối xứng thì đã biết còn bđt thuần nhất là khi thay $a=tx,b=ty,c=tz$ thì bđt không thay đổi

ví dụ như CM $\frac{8(a^2+b^2+c^2)}{ab+bc+ca}+\frac{27(a+b)(b+c)(c+a)}{(a+b+c)^3}\geq 16$$(a,b,c>0)$

khi đặt $a=tx,b=ty,c=tz$ thì bđt vẫn không thay đổi

khi mới học cái chuẩn hóa này thì nghĩ tới câu hỏi sao lại có thể đặt $a+b+c=3,abc=1,...$ ta hiểu đơn giản như sau

nếu đặt $x=\frac{a}{a+b+c},y=\frac{b}{a+b+c},z=\frac{c}{a+b+c}\Rightarrow x+y+z=1$ và khi thay vào bđt ắt hẳn bđt sẽ không đổi

nếu đặt $x=\frac{a}{\sqrt[3]{abc}},y=\frac{b}{\sqrt[3]{abc}},z=\frac{c}{\sqrt[3]{abc}}\Rightarrow xyz=1$

và mấy cách chuẩn hóa khác cũng tương tự như vậy

 

NTP

Nhận định này của em sai nhé.




#531773 $\sum \frac{a^{2}+b^{2}}{a+b}\leq 3\frac{a^{2}+b^{2}...

Gửi bởi Nguyenhuyen_AG trong 04-11-2014 - 16:26

Bất đẳng thức này đúng theo bất đẳng thức Schur và Cauchy-Schwarz.




#527119 CMR: $2ab(a+b)+2bc(b+c)+2ca(c+a)\geq a^3+b^3+c^3+9abc$

Gửi bởi Nguyenhuyen_AG trong 04-10-2014 - 15:30

Đổi biến $a=x+y,\,b==y+z,\,c=z+x$ thì ta sẽ được bất đẳng thức Schur bậc 3.




#527058 $(a + b)(b + c)(c + a) \geq 2(a + b + c + 1)$

Gửi bởi Nguyenhuyen_AG trong 03-10-2014 - 22:02

Cho abc = 1

a) $(a + b)(b + c)(c + a) \geq 2(a + b + c + 1)$

b) $(a + b)(b + c)(c + a) \geq 4(a + b + c - 1)$

Câu a) chính là bài 3 trong đề thi APMO 1998, có một lời giải bằng Cauchy-Schwarz cho bài này. Còn câu b) là đề MOSP năm 2 nghìn lẻ mấy không nhớ. =)) Câu này có khá nhiều cách chứng minh. Xin trình bày một chứng minh bằng AM-GM.

 

Ta viết bất đẳng thức lại dưới dạng thuần nhất

\[\frac{(a + b)(b + c)(c + a)}{abc} \geqslant 4\left(\frac{a + b + c}{\sqrt[3]{abc}} - 1\right),\]

\[\frac{(a + b)(b + c)(c + a)}{abc} +4 \geqslant \frac{4(a + b + c)}{\sqrt[3]{abc}}.\]

Áp dụng bất đẳng thức AM-GM, ta có

\[\begin{aligned} \frac{(a + b)(b + c)(c + a)}{abc} +4 &=\frac{(a + b)(b + c)(c + a)}{2abc}+\frac{(a + b)(b + c)(c + a)}{2abc}+4 \\& \geqslant 3\sqrt[3]{\frac{(a + b)^2(b + c)^2(c + a)^2}{a^2b^2c^2}}.\end{aligned}\]

Vậy ta cần phải chứng minh

\[3\sqrt[3]{\frac{(a + b)^2(b + c)^2(c + a)^2}{a^2b^2c^2}} \geqslant \frac{4(a + b + c)}{\sqrt[3]{abc}},\]

lập phương 2 vế ta được

\[27(a + b)^2(b + c)^2(c + a)^2 \geqslant 64abc(a + b + c)^3.\]

Bất đẳng thức này đúng theo hai bất đẳng thức cơ bản sau:

\[(a+b)(b+c)(c+a) \ge \geqslant \frac{8(a+b+c)(ab+bc+ca)}{9},\]

\[(ab+bc+ca)^2 \ge 3abc(a+b+c).\]

Đẳng thức xảy ra khi $a=b=c.$ Bài toán được chứng minh




#519336 CMR:$\frac{ab}{3a^2+b^2}+\frac{bc...

Gửi bởi Nguyenhuyen_AG trong 13-08-2014 - 19:47

Gợi ý: Tách $3a^2+b^2=2a^2+(a^2+b^2)$ rồi sử dụng bất đẳng thức AM-GM.

 

Bạn hãy đặt bút suy nghĩ thêm tí nữa, chưa gì đã bảo không được rồi.




#519314 CMR:$\frac{ab}{3a^2+b^2}+\frac{bc...

Gửi bởi Nguyenhuyen_AG trong 13-08-2014 - 17:49

Gợi ý: Tách $3a^2+b^2=2a^2+(a^2+b^2)$ rồi sử dụng bất đẳng thức AM-GM.




#519313 $\frac{1}{a+b}+\frac{1}{b+c...

Gửi bởi Nguyenhuyen_AG trong 13-08-2014 - 17:47

Cho $a,b,c\geq 0$ thỏa mãn $ab+bc+ca=1$. CMR:
$$\frac{1}{a+b}+\frac{1}{b+c}+\frac{1}{c+a}-\frac{1}{a+b+c}\geq 2$$

Bất đẳng thức cần chứng minh tương đương với

$$\frac{1}{a+b}+\frac{1}{b+c}+\frac{1}{c+a}\geq 2+\frac{1}{a+b+c},$$

$$\frac{a+b+c}{a+b}+\frac{a+b+c}{b+c}+\frac{a+b+c}{c+a}\geq 2(a+b+c)+1,$$

$$\frac{c}{a+b}+\frac{a}{b+c}+\frac{b}{c+a}+2\geq 2(a+b+c).$$

Áp dụng bất đẳng thức Cauchy-Schwarz và AM-GM ta có

$$\frac{c}{a+b}+\frac{a}{b+c}+\frac{b}{c+a}+2\ge \frac{(a+b+c)^2}{2(ab+bc+ca)}+2=\frac{(a+b+c)^2}{2}+2 \geq 2(a+b+c).$$

Đẳng thức xảy ra khi $a=b,c=0$ cùng các hoán vị.




#508579 $\frac{a+b+c}{3}-\sqrt[3]{abc}...

Gửi bởi Nguyenhuyen_AG trong 23-06-2014 - 14:15

Cho a,b,c là các số thực dương .CMR : 

 

      $$\frac{a+b+c}{3}-\sqrt[3]{abc}\leq max\left \{ (\sqrt{a}-\sqrt{b})^2,(\sqrt{b}-\sqrt{c})^2,(\sqrt{c}-\sqrt{a})^2 \right.\left. \right \}$$

Hãy tìm hằng số $k$ tốt nhất của bất đẳng thức

\[\frac{a+b+c}{3}-\sqrt[3]{abc}\leq k\cdot\max\left \{ (\sqrt{a}-\sqrt{b})^2,\,(\sqrt{b}-\sqrt{c})^2,\,(\sqrt{c}-\sqrt{a})^2\right\},\]

với $a,\,b,\,c$ là các số thực không âm.




#489199 Bạn đã tìm lời giải như thế nào ?

Gửi bởi Nguyenhuyen_AG trong 28-03-2014 - 16:19

Mọi người tiếp tục nhé. Đây là bài thi quốc gia năm 2008.

 

$\boxed{\text{Bài 7.}}$ Với $x,\,y,\,z$ là ba số thực không âm đôi một khác nhau, hãy chứng minh

\[(xy+yz+zx)\left [\frac{1}{(x-y)^2}+\frac{1}{(y-z)^2}+\frac{1}{(z-x)^2} \right ] \ge 4.\]




#485266 Bạn đã tìm lời giải như thế nào ?

Gửi bởi Nguyenhuyen_AG trong 01-03-2014 - 11:52


 Tương tự ta có bài toán sau:
Cho $a,b,c$ thực dương thỏa $2a+2b+c^2=14$. Tìm giá trị nhỏ nhất của $$P=a^3+2b^2+2c^3$$ (Trích đề thi thử Moon.vn lần 2-2014)
 

Spoiler

 

Hoàn toàn tương tự. Ta giả sử $a=x,\;b=y,\;c=z$ và sử dụng bất đẳng thức AM-GM để tạo ra $a,\;b$ và $b^2.$ Ta có

\[a^3+2b^2+2c^3 \ge \left ( 3ax^2+4by+3c^2z \right )-\left ( 2x^3+2y^2+z^3 \right ).\]

Để sử dụng được điều kiện thì ta cần tách

\[3ax^2+4by+3c^2z=2a\cdot \frac{3x^2}{2}+2b\cdot2y+c^2\cdot3z.\]

Như vậy ta cần chọn $x,\;y,\;z$ thoả mãn điều kiện

\[\left\{\begin{matrix}&\dfrac{3x^2}{2}=2y=3z \\& 2x+2y+z^2=14\end{matrix}\right.\] 

Giải hệ này ta được $x=2,\;y=3,\;z=2$ và có được $P_{\min} = 34.$




#485124 Bạn đã tìm lời giải như thế nào ?

Gửi bởi Nguyenhuyen_AG trong 28-02-2014 - 15:14

Tình cờ thấy được bài này trên diễn đàn mọi người cùng phân tích thử xem nhé. 

 

$\boxed{\text{Bài 6.}}$ Cho hai số dương $a,\;b$ thỏa mãn điều kiện $a^2+b^2=5.$ Hãy tìm giá trị nhỏ nhất của biểu thức

\[P=a^3+b^6.\]

 

Bài này có ý tưởng AM-GM rất rõ ràng, giả thiết của bài toán chứa biểu thức có bậc nhỏ hơn bậc của $P,$ và việc sử dụng bất đẳng thức AM-GM để hạ bậc là một ý tưởng rất tự nhiên. Tuy nhiên, chúng ta lại không biết là dấu bằng của bài toán sẽ xảy ra khi nào nên việc chọn hệ số để sử dụng bất đẳng thức AM-GM cũng khá vất vả, vì vậy ta sẽ sử dụng kỹ thuật chọn dấu bằng giả định.

 

Cụ thể, ta sẽ giả sử đẳng thức của bài toán sẽ đạt được tại $a=x,\;b=y$ với $x,\;y$ là những hằng số. Khi đó theo bất đẳng thức AM-GM, ta có

\[a^3+a^3+x^3 \ge 3\sqrt[3]{a^3\cdot a^3 \cdot x^3}=3a^2\cdot x, \quad (1)\]

tức là

\[2a^3+x^3 \ge 3a^2\cdot x,\]

\[a^3\ge \frac{3a^2\cdot x-x^3 }{2}.\]

Hoàn toàn tườn tự, ta có 

\[b^6\ge 3b^2\cdot y^4-2y^6\]

Như vậy, ta có 

\[ a^3+b^6 \ge 3b^2\cdot y^4-2y^6+\frac{3a^2\cdot x}{2}-\frac{x^3}{2}=3\left ( b^2\cdot y^2+a^2\cdot \frac{x}{2} \right )-2y^6-\frac{x^3}{2}. \quad (2)\]

Ta cần chọn $x,\;y$ sao cho trong $(2)$ ta có thể sử sụng được giả thiết $a^2+b^2=5$ đồng thời cũng thoả mãn điều kiện của bait tóan, tức $x,\;y$ phải thoả mãn hệ  phương trình

\[\left\{\begin{matrix}x^2+y^2&=5 \\y^2&=\frac{x}{2}\end{matrix}\right.\]

Giải hệ này ta được $x=2,\;y=1$ và suy ra $P_{\min} =9.$

 

Nhận xét: Có một số bạn sẽ đặt câu hỏi rằng tại sao lại dùng $2$ đại lượng $a^3$ kết hợp với $x^3$ để sử dụng bất đẳng thức AM-GM cho ba số. Để ý rằng điều kiện của bài toán cho ta $a^2+b^2=5$ nên chúng ta phải cố gắng đánh giá $a^3$ sao cho xuất hiện được $a^2$ và $b^6$ sao cho xuất hiện được $b^2$ để áp dụng giả thiết $a^2+b^2=5.$ Điều này lý giải cho việc áp dụng bất đẳng thức AM-GM như trên.




#483805 Bạn đã tìm lời giải như thế nào ?

Gửi bởi Nguyenhuyen_AG trong 18-02-2014 - 11:52

Tình cờ thấy được bài này trên diễn đàn mọi người cùng phân tích thử xem nhé. 

 

$\boxed{\text{Bài 6.}}$ Cho hai số dương $a,\;b$ thỏa mãn điều kiện $a^2+b^2=5.$ Hãy tìm giá trị nhỏ nhất của biểu thức

\[P=a^3+b^6.\]




#477645 Bạn đã tìm lời giải như thế nào ?

Gửi bởi Nguyenhuyen_AG trong 17-01-2014 - 01:49



Mọi người tiếp tục thảo luận nhé. Lúc nãy qua bên Mathscope, thấy có một bạn post bài này.

 

Bài 3. Cho $a,\;b,\;c$ là các số thực thay đổi tùy ý thỏa mãn $ab+bc+ca=1.$ Hãy tìm giá trị nhỏ nhất của biểu thức sau

\[P=a^2+2b^2+5c^2.\]

 

Mấy tuần này mình bận thi học kỳ nên không vào diễn đàn thường xuyên được. Giờ mình sẽ giải bất đẳng thức này, chúng ta sẽ giải quyết bài toán tổng quát luôn

 

Với $a,\;b$ là hai số thực dương cho trước và $x,\;y,\;z$ là các số thực thay đổi thỏa mãn điều kiện $xy+yz+zx=1.$
 
$(a)$ Chứng minh rằng phương trình.
$$2t^3+(1+a+b)t^2-ab=0,$$
có nghiệm dương duy nhất.
 
$(b)$ Tìm giá trị nhỏ nhất của biểu thức
$$P=x^2+ay^2+bz^2. \quad (5)$$
Phần $(a)$ khá đơn giản, có thể tự chứng mình lấy bằng cách sử dụng đạo hàm. Ở đây ta chỉ quan tâm đến phần $(b)$ của bài toán.
 
Ta thấy biểu thức $P$ của bài toán có dạng $A^2+B^2+C^2,$ rất giống với bất đẳng thức Cauchy-Schwaz dạng phân thức là
$$\frac{A^2}{u}+\frac{B^2}{v}+\frac{C^2}{w} \ge \frac{(A+B+C)^2}{u+v+w}.$$
Điều này gợi ý cho ta sử dụng bất đẳng thức Cauchy-Schwarz để chứng minh bài toán.
 
Tuy nhiên, vế trái của ta đã có dạng tổng các bình phương rồi, nhưng biểu thức bình phương ở vế phải sẽ như thế nào ? Bây giờ ta sẽ tìm cách tạo ra bình phương ở vế phải bằng cách sử dụng giả thiết $xy+yz+zx=1.$
 
Trước hết, ta sẽ thuần nhất bài toán để đưa bài toán về đúng với bản chất của nó. Giả sử $P$ đạt giá trị nhỏ nhất bằng $2t \;(t>0)$ (tại sao là $2t$ mà không phải là số nào khác ? t chẳng hạn) Khi đó ta có bất đẳng thức sau
$$x^2+ay^2+bz^2 \ge 2t.$$
Hay viết dưới dạng thuần nhất là 
$$x^2+ay^2+bz^2 \ge 2t(xy+yz+zx). \quad (5.1)$$
Đến đây thì ý tưởng tạo bình phương bên vế phải đã xuất hiện, vì chỉ cần thêm vào đại lượng $t(x^2+y^2+z^2)$ ta sẽ có ngay $2t(xy+yz+zx)+t(x^2+y^2+z^2)=t(x+y+z)^2.$ Đến đây chắc hẳn ta đã thấy được tác dụng của việc chọn giá trị nhỏ nhất của $P$ là $2t$ thay vì là một số khác. Như là $t$ chẳng hạn, vì khi đó ta phải cộng thêm vào một đại lượng $\frac{t(x^2+y^2+z^2)}{2}$ là một phân thức thay vì dạng đơn giản hơn $t(x^2+y^2+z^2).$
 
Với ý tương trên, ta có thế viết $(5.1)$ lại như sau
$$x^2+ay^2+bz^2+t(x^2+y^2+z^2) \ge 2t(xy+yz+zx)+t(x^2+y^2+z^2),$$
tương đương với
$$(t+1)x^2+(t+a)y^2+(t+b)z^2\ge t(x+y+z)^2.$$
Sử dụng bất đẳng thức Cauchy-Schwarz, ta có 
$$(t+1)x^2+(t+a)y^2+(t+b)z^2 \ge \frac{(x+y+z)^2}{\frac{1}{t+1}+\frac{1}{t+a}+\frac{1}{t+b}}.$$
Ta cần cần chọn $t$ sao cho
$$\frac{(x+y+z)^2}{\frac{1}{t+1}+\frac{1}{t+a}+\frac{1}{t+b}}=\frac{(x+y+z)^2}{\frac{1}{t}},$$
hay là
$$\frac{1}{t+1}+\frac{1}{t+a}+\frac{1}{t+b}=\frac{1}{t},$$
quy đồng và thu gọn, ta được
$$2t^3+(1+a+b)t^2-ab=0.$$
Kết hợp với câu $(a)$ ta suy ra $P$ đạt giá trị nhỏ nhất là $2t$ với $t$ là nghiệm dương của đa thức
$$f(t)=2t^3+(1+a+b)t^2-ab.$$
Đẳng thức xảy ra khi và chỉ khi $(t+1)x=(t+a)y=(t+b)z.$



#477643 $\frac{1}{a+b}+\frac{1}{b+c...

Gửi bởi Nguyenhuyen_AG trong 17-01-2014 - 01:32



Cho a,b,c> o thoả mãn: a+b+c= ab+bc+ca

CMR:$\frac{1}{a+b}+\frac{1}{b+c}+\frac{1}{c+a}\leqslant \frac{3}{2}$

 

Bất đẳng thức cần chứng minh được viết lại dưới dạng thuần nhất là

\[\frac{1}{a+b}+\frac{1}{b+c}+\frac{1}{c+a}\leqslant \frac{3}{2}\cdot \frac{a+b+c}{ab+bc+ca}.\]

Chú ý rằng 

\[\frac{ab+bc+ca}{a+b}=\frac{c(a+b)+ab}{a+b}=c+\frac{ab}{a+b},\]

nên bất đẳng thức được rút gọn lại nhưu sau

\[\frac{ab}{a+b}+\frac{bc}{b+c}+\frac{ca}{c+a} \le \frac{a+b+c}{2}.\]

Bất đẳng thức này đúng theo bất đẳng thức cơ bản $(x+y)^2 \ge 4xy.$

 

Ngoài ra bài toán còn được chứng minh bằng cách sử dụng hai đánh giá quen thuộc $(a+b+c)^2 \ge 3(ab+bc+ca)$ và $(a+b)(b+c)(c+a) \ge \frac{8}{9}\cdot (a+b+c)(ab+bc+ca).$

 

Từ bài này ta suy ra được bất đẳng thức khá hay sau đây (nếu mình nhớ đúng đề :v)

 

\[\frac{1}{a+b+2}+\frac{1}{b+c+2}+\frac{1}{c+a+2}\leqslant \frac{3}{4},\]

với $a,\;b,\;c$ cùng điều kiện trên và $ab+bc+ca=3.$




#477631 CMR: $\sum \sqrt{\frac{a}{b+c}...

Gửi bởi Nguyenhuyen_AG trong 16-01-2014 - 22:23


Cho a, b, c không âm và không có hai số nào đồng thời bằng 0. CMR: $\sqrt{\frac{a}{b+c}}+\sqrt{\frac{b}{c+a}}+\sqrt{\frac{c}{a+b}}\geq 2\sqrt{1+\frac{abc}{(a+b)(b+c)(c+a)}}$

 

Bất đẳng thức này là của Vũ Đình Quý, khi ta nhân 2 vế của bất đẳng thức này cho $(a+b)(b+c)(c+a)$ và chú ý rằng $(a+b)(b+c)(c+a)=(a+b+c)(ab+bc+bc+ca)-abc,$ ta có thể viết bất đẳng thức trên lại như sau

\[\sum \sqrt{a(a+b)(a+c)} \ge 2 \sqrt{(a+b+c)(ab+bc+ca)},\]

hay là

\[\sum \sqrt{a^3+a(ab+bc+ca)} \ge 2 \sqrt{(a+b+c)(ab+bc+ca)}.\]

Chuẩn hóa cho $ab+bc+ca=1,$ bất đẳng thức trở thành

\[\sum \sqrt{a^3+a} \ge 2 \sqrt{a+b+c}.\]

Đây chính là đề thi Iran TST 2008. Có rất nhiều lời giải cho bài toán này, bạn có thể tham khảo ở đây. http://www.artofprob...p?f=52&t=206627